LSAT and Law School Admissions Forum

Get expert LSAT preparation and law school admissions advice from PowerScore Test Preparation.

 Administrator
PowerScore Staff
  • PowerScore Staff
  • Posts: 8927
  • Joined: Feb 02, 2011
|
#66023
Complete Question Explanation

The correct answer choice is (E).

Answer choice (A):

Answer choice (B):

Answer choice (C):

Answer choice (D):

Answer choice (E): This is the correct answer choice.


This explanation is still in progress. Please post any questions below!
 gomezcb
  • Posts: 1
  • Joined: Jul 09, 2020
|
#77014
Hello! Can you please provide an explanation for this question? I'm having trouble understanding how Answer E and B would not lead to the same outcome.
 Jeremy Press
PowerScore Staff
  • PowerScore Staff
  • Posts: 1000
  • Joined: Jun 12, 2017
|
#77094
Hi Gomez,

Sure!

The reason answer choice B doesn't lead to a complete solution is that the rule about H and S only means that if you have S shown in a week, then H must also be shown in that week. But (since it's the necessary condition in the rule) H could be shown in a given week without S. So just placing H in two of the weeks (weeks 2 and 4, since H can't go first, and the two H's can't be consecutive) doesn't tell us for sure which of P, R, or S is shown with it. Maybe S is shown with it both weeks. Or maybe S is shown with H in one week, and R is shown with H in another week. That uncertainty means I can't get to a complete solution from answer choice B

BUT, in answer choice E, when we place S in two weeks, those two weeks have to be weeks 2 and 4 (from our Global sketch, S cannot be shown in week 1). Once we place S in those weeks, H is also shown in those same two weeks, because the rule is that if (any week in which) S is shown, then H must be shown in that same week. With H definitely placed in week 2, I must be in week 1 (from the last rule in the game). G must be shown at least once, and the only available week for it to be shown is now week 3. P must be shown at least once, but not with G, so P must now be shown in week 1. That leaves only one slot open (in week 3) for R, which must be shown then because it also must be shown at least once. That gives us our full and complete solution.

Hope this helps!

Jeremy
 theamazingrace
  • Posts: 59
  • Joined: Oct 17, 2020
|
#80403
Hi, I tried answer D and it worked but only when I put R in 2 and 4 and it did not work when I placed R in 1 and 4. So, when a question says "completely determine" does that mean it has to work for all possible placements?

Thanks
 Paul Marsh
PowerScore Staff
  • PowerScore Staff
  • Posts: 290
  • Joined: Oct 15, 2019
|
#80632
Hi theamazingrace! Sometimes you'll get a question like this that asks, "the game is completely determined if which of the following is true?" We call these Justify Logic Game questions. You asked:
So, when a question says "completely determine" does that mean it has to work for all possible placements?
In a word - no. Instead, what these questions are basically asking is: which answer choice, if it were true, would allow us to figure out where every single variable in the game for sure goes? So our right answer will leave us with just one possible scenario for the entire game.

So like Jeremy explained above, if we know that S goes twice, we can figure out where every single variable in the game goes. We've "completely determined" the game.

On the other hand, if (as answer choice D suggests) we know that R goes twice, we can't figure out where every single variable in the game goes. In fact, we can't even figure out where R goes! (It could go in 2 and 4 like you said; but it could also go in say, 1 and 3).

These Justify questions can be tricky. Oftentimes they're worth doing near the end of that particular game, so that you have a better sense of which variable placements really allow all the other variables to fall into place.

Hope that helps!
 startedfromKZnowwehere
  • Posts: 7
  • Joined: Jul 11, 2021
|
#89246
Hi there!

I understand the answer, but I basically just guessed and moved on for this one because I didn't want to spend time trying everything with hypotheticals.

Is the recommended plan of attack for Justify questions to find limiting variables (such as S) and then start your hypotheticals from there to see if it works? Like an educated guess, sort of?

Thanks!
User avatar
 atierney
PowerScore Staff
  • PowerScore Staff
  • Posts: 215
  • Joined: Jul 06, 2021
|
#89347
Hi,

Without going into the specifics of this particular game, the ease with which you are able to answer these justify the conclusion questions on the LG section, typically, will depend upon the comprehensiveness of premises that you have already determined via inferences derived from the rules themselves or from their application on prior questions. In other words, essentially, the answer to this type question depends upon how many "degrees of freedom" (my term) exist within your already-constructed global template of the game itself. To the extent that you've basically determined there is essentially one degree of freedom (two solutions to the game, hypothetically speaking), then solving the game is simply a matter of eliminating this particular uncertainty (notice the similarity, in this regard, to LR Justify questions). Therefore, in answer to your question, if you're looking for a way to solve this type of question in particular, without the requisite "grunt work" of going through hypotheticals in your mind (which is not necessarily a bad scratch, I must say), then look no forward then the beautiful diagramming that you are doing for each logic game in general, and the inferences the follow therefrom.

Now, having said that, that's from three thousand feet up, and ultimately, it really is a matter of balancing the time you have for any given game. However, to the extent that you fall on the smarter side of hard-smart work continuum, then I think the general solution to your problems will be to emphasize diagramming the games as much as possible, and really working on making inferences on the basis of the interplay between rules. This is definitely something that we emphasize at PowerScore, and I believe if you practice your diagramming ability, it will definitely assist in visualizing the requisite inferences that may lie beyond the naked mind's eye.

Let me know if you have further questions on this. Like I said, this is a general answer to a question the answer of which demands such breadth (enjoy the view!).
 Ashley.mly
  • Posts: 3
  • Joined: Sep 11, 2021
|
#93170
Can someone please help me understand why (A) Gold is shown twice is incorrect?

Hypotheticals:
GIGH IGHG
RPRS PRSR

It appears that if G is shown twice, G can either go 1 and 3 or 2 and 4. If this is the case, then the places in which I and H can go are completely determined. Following, because there must be at least one of each painting shown in a week, the S must be paired with the H, leaving I with P because of the no consecutive rule.

Although (E) does completely determine the schedule, I cannot see why (A) is incorrect; it seems to completely determine as well.. any help is appreciated!
 Ashley.mly
  • Posts: 3
  • Joined: Sep 11, 2021
|
#93171
Whoops, I've found the hypothetical: GIHG
(R/S)PSR

The first watercolor painting can be either R or S. Thus, the order is not completely determined. In case this post can be helpful to another student, will leave it here. Happy studying everyone! =]


Ashley.mly wrote: Sun Jan 09, 2022 5:25 pm Can someone please help me understand why (A) Gold is shown twice is incorrect?

Hypotheticals:
GIGH IGHG
RPRS PRSR

It appears that if G is shown twice, G can either go 1 and 3 or 2 and 4. If this is the case, then the places in which I and H can go are completely determined. Following, because there must be at least one of each painting shown in a week, the S must be paired with the H, leaving I with P because of the no consecutive rule.

Although (E) does completely determine the schedule, I cannot see why (A) is incorrect; it seems to completely determine as well.. any help is appreciated!
 Robert Carroll
PowerScore Staff
  • PowerScore Staff
  • Posts: 1774
  • Joined: Dec 06, 2013
|
#93380
Ashley,

Even in your first post, if G's being shown twice is compatible with two different solutions, then G's being shown twice will not completely determine the schedule of the paintings.

Robert Carroll

Get the most out of your LSAT Prep Plus subscription.

Analyze and track your performance with our Testing and Analytics Package.